(Created page with "Category:ECE Category:QE Category:problem solving <center> <font size= 4> ECE Ph.D. Qualifying Exam </font size> <font size= 4> Auto...")
 
Line 18: Line 18:
 
----
 
----
 
===Solution===
 
===Solution===
Let <math>t_1=x_1-1</math>, <math>t_2=x_2+1</math><br>
+
Let <math>t_1=x_1-2</math>, <math>t_2=x_2+1</math><br>
so that <math>g(t_1,t_2)=\dfrac{1}{t_1^2+t_2^2+3}</math>
+
so that <math>g(t_1,t_2)=\dfrac{1}{t_1^2+t_2^2+3}|t_1=0,t_2</math> would have some convex property with <math>f(x_1,x_2)=\dfrac{1}{(x_1-2)^2+(x_2+1)^2+3}|x_1=2,x_1=-1</math><br>
 
----
 
----
 
[[QE2016_AC-3_ECE580|Back to QE AC question 3, August 2016]]
 
[[QE2016_AC-3_ECE580|Back to QE AC question 3, August 2016]]
  
 
[[ECE_PhD_Qualifying_Exams|Back to ECE Qualifying Exams (QE) page]]
 
[[ECE_PhD_Qualifying_Exams|Back to ECE Qualifying Exams (QE) page]]

Revision as of 21:59, 18 February 2019


ECE Ph.D. Qualifying Exam

Automatic Control (AC)

Question 3: Optimization

August 2016 Problem 3


Solution

Let $ t_1=x_1-2 $, $ t_2=x_2+1 $
so that $ g(t_1,t_2)=\dfrac{1}{t_1^2+t_2^2+3}|t_1=0,t_2 $ would have some convex property with $ f(x_1,x_2)=\dfrac{1}{(x_1-2)^2+(x_2+1)^2+3}|x_1=2,x_1=-1 $


Back to QE AC question 3, August 2016

Back to ECE Qualifying Exams (QE) page

Alumni Liaison

Correspondence Chess Grandmaster and Purdue Alumni

Prof. Dan Fleetwood